LSAT and Law School Admissions Forum

Get expert LSAT preparation and law school admissions advice from PowerScore Test Preparation.

 Administrator
PowerScore Staff
  • PowerScore Staff
  • Posts: 8917
  • Joined: Feb 02, 2011
|
#38954
Complete Question Explanation
(The complete setup for this game can be found here: lsat/viewtopic.php?t=14969)

The correct answer choice is (C)

This question establishes that exactly two of the applicants are evaluated by T. Since S must evaluate more applicants than T, it follows that S must evaluate three applicants, and the 1-3-2-1 distribution applies:
PT71_D13 LG Explanations_Game #2_#8_diagram 1.png
According to our distribution, U evaluates only one applicant (i.e. G). Also, from our initial setup, we know that K must be evaluated by either R or T. A distribution where T evaluates two applicants forces K to be evaluated by R, because the officer who evaluates K cannot evaluate anyone else (fourth rule):
PT71_D13 LG Explanations_Game #2_#8_diagram 2.png
Now, recall our inference that F and L must be evaluated by either R or S (if F and L were evaluated by T, this would force H, I and M to be evaluated by the same officer, in violation of the third rule). In this case, since R must evaluate K, and K only, we can infer that F and L must be evaluated by S:
PT71_D13 LG Explanations_Game #2_#8_diagram 3.png
Among the remaining variables (H, I, and M), I cannot be evaluated by the same officer who evaluates H or M. Since T must evaluate two applicants, it follows that T cannot evaluate I. Therefore, T must evaluate H and M, while S evaluates I:
PT71_D13 LG Explanations_Game #2_#8_diagram 4.png
Our local diagram immediately shows that answer choice (C) cannot be true, and is therefore the correct answer to this Must Be True, EXCEPT question. Although the stem is asking us to identify what need not to be true, an answer choice that cannot be true would also satisfy this condition.
You do not have the required permissions to view the files attached to this post.
 markjatwater
  • Posts: 3
  • Joined: Jun 24, 2017
|
#36313
I'm confused on why E was right.

If exactly 2 applications are done by T and S needs more applications than T than shouldn't it be

R: k
S: i,f,l
T: h,m
U: G

Thanks!
 Luke Haqq
PowerScore Staff
  • PowerScore Staff
  • Posts: 742
  • Joined: Apr 26, 2012
|
#36451
Hi Mark!

I think the there might be a typo or error, if you see somewhere stating (E) is the right answer for this one. The correct answer should be (C).

And you've diagrammed everything correctly, so no problem there. Indeed, your diagram shows why (A), (B), (D), and (E) all must be true, and further, shows that F must be reviewed by Smith, leaving (C) as the correct answer.

Hope that helps!
 rwraulynaitis
  • Posts: 27
  • Joined: May 06, 2020
|
#76649
Hello,

Can you explain the inference that the FL block must be evaluated by R or S? I think a hypothetical I made follows all of the rules and places the FL block with U:


R: K
S: H M
T: I
U: F L G


Thanks!
 Adam Tyson
PowerScore Staff
  • PowerScore Staff
  • Posts: 5153
  • Joined: Apr 14, 2011
|
#76682
I think you missed the requirement of this local question, rwraulynaitis, which is that T must evaluate exactly two of the applications. In your hypo, T only does one! Be sure to follow all the original rules AND the restriction presented in the local question.

Get the most out of your LSAT Prep Plus subscription.

Analyze and track your performance with our Testing and Analytics Package.